How should I solve this triple integral?Finding the Limits of the Triple Integral (Spherical Coordinates)How to setup/evaluate a triple integral to show an interesting result in physics?Triple Integrals in Spherical Coordinates where (z-2)^2Triple integral in spherical / cylindrical coordinates - where's the error? Exercise checktriple integral over spherical region and of a shifted sphere integrandTriple Integral in spherical coordinatesCalculating triple integral over a region GTrying to evaluate this triple integral?Triple Integrals in Spherical Coordinates, problem with boundariesComputing Triple Integral Using Spherical Coordinates

How could our ancestors have domesticated a solitary predator?

Does splitting a potentially monolithic application into several smaller ones help prevent bugs?

Could you please stop shuffling the deck and play already?

If the Captain's screens are out, does he switch seats with the co-pilot?

Should QA ask requirements to developers?

Could a cubesat be propelled to the moon?

PTIJ: Why can't I eat anything?

The bar has been raised

What do you call the air that rushes into your car in the highway?

Aliens englobed the Solar System: will we notice?

Placing subfig vertically

Unreachable code, but reachable with exception

Solving "Resistance between two nodes on a grid" problem in Mathematica

Virginia employer terminated employee and wants signing bonus returned

Space in array system equations

Can't find the Shader/UVs tab

Why don't MCU characters ever seem to have language issues?

Fourth person (in Slavey language)

Why the color red for the Republican Party

Algorithm to convert a fixed-length string to the smallest possible collision-free representation?

Why does Captain Marvel assume the planet where she lands would recognize her credentials?

Single word request: Harming the benefactor

String reversal in Python

Is having access to past exams cheating and, if yes, could it be proven just by a good grade?



How should I solve this triple integral?


Finding the Limits of the Triple Integral (Spherical Coordinates)How to setup/evaluate a triple integral to show an interesting result in physics?Triple Integrals in Spherical Coordinates where (z-2)^2Triple integral in spherical / cylindrical coordinates - where's the error? Exercise checktriple integral over spherical region and of a shifted sphere integrandTriple Integral in spherical coordinatesCalculating triple integral over a region GTrying to evaluate this triple integral?Triple Integrals in Spherical Coordinates, problem with boundariesComputing Triple Integral Using Spherical Coordinates













3












$begingroup$


Hi everybody I have a triple integral I can't solve:




$$iiint sqrt x^2+y^2+z^2 ,dx ,dy ,dz $$




Which the region is between $z=sqrt x^2+y^2$ and $z=4$ .
The question says after using the spherical coordinates the answer is:




$$aLargeint _0^pi/4 frac(sin phi)dphicos^4phi $$




So I've used the spherical coordinates $0<r<4$ and $ 0<theta<2pi$ and $0<phi<fracpi2$. The things I don't get is first why is $phi$ varying between 0 and $fracpi4$ instead of $fracpi2$? And secondly where did that $cos^4phi$ came from?



By the way the question asks for the value of $a$.










share|cite|improve this question









New contributor




khoshrang is a new contributor to this site. Take care in asking for clarification, commenting, and answering.
Check out our Code of Conduct.







$endgroup$











  • $begingroup$
    EDIT: ok now i understand the ϕ<π/4 it's because if we put x=0 in z=√ x^2+y^2we will get z=+y and z = -y which explains the ϕ bounds
    $endgroup$
    – khoshrang
    2 days ago















3












$begingroup$


Hi everybody I have a triple integral I can't solve:




$$iiint sqrt x^2+y^2+z^2 ,dx ,dy ,dz $$




Which the region is between $z=sqrt x^2+y^2$ and $z=4$ .
The question says after using the spherical coordinates the answer is:




$$aLargeint _0^pi/4 frac(sin phi)dphicos^4phi $$




So I've used the spherical coordinates $0<r<4$ and $ 0<theta<2pi$ and $0<phi<fracpi2$. The things I don't get is first why is $phi$ varying between 0 and $fracpi4$ instead of $fracpi2$? And secondly where did that $cos^4phi$ came from?



By the way the question asks for the value of $a$.










share|cite|improve this question









New contributor




khoshrang is a new contributor to this site. Take care in asking for clarification, commenting, and answering.
Check out our Code of Conduct.







$endgroup$











  • $begingroup$
    EDIT: ok now i understand the ϕ<π/4 it's because if we put x=0 in z=√ x^2+y^2we will get z=+y and z = -y which explains the ϕ bounds
    $endgroup$
    – khoshrang
    2 days ago













3












3








3





$begingroup$


Hi everybody I have a triple integral I can't solve:




$$iiint sqrt x^2+y^2+z^2 ,dx ,dy ,dz $$




Which the region is between $z=sqrt x^2+y^2$ and $z=4$ .
The question says after using the spherical coordinates the answer is:




$$aLargeint _0^pi/4 frac(sin phi)dphicos^4phi $$




So I've used the spherical coordinates $0<r<4$ and $ 0<theta<2pi$ and $0<phi<fracpi2$. The things I don't get is first why is $phi$ varying between 0 and $fracpi4$ instead of $fracpi2$? And secondly where did that $cos^4phi$ came from?



By the way the question asks for the value of $a$.










share|cite|improve this question









New contributor




khoshrang is a new contributor to this site. Take care in asking for clarification, commenting, and answering.
Check out our Code of Conduct.







$endgroup$




Hi everybody I have a triple integral I can't solve:




$$iiint sqrt x^2+y^2+z^2 ,dx ,dy ,dz $$




Which the region is between $z=sqrt x^2+y^2$ and $z=4$ .
The question says after using the spherical coordinates the answer is:




$$aLargeint _0^pi/4 frac(sin phi)dphicos^4phi $$




So I've used the spherical coordinates $0<r<4$ and $ 0<theta<2pi$ and $0<phi<fracpi2$. The things I don't get is first why is $phi$ varying between 0 and $fracpi4$ instead of $fracpi2$? And secondly where did that $cos^4phi$ came from?



By the way the question asks for the value of $a$.







integration spherical-coordinates multiple-integral






share|cite|improve this question









New contributor




khoshrang is a new contributor to this site. Take care in asking for clarification, commenting, and answering.
Check out our Code of Conduct.











share|cite|improve this question









New contributor




khoshrang is a new contributor to this site. Take care in asking for clarification, commenting, and answering.
Check out our Code of Conduct.









share|cite|improve this question




share|cite|improve this question








edited 2 days ago









Max

629317




629317






New contributor




khoshrang is a new contributor to this site. Take care in asking for clarification, commenting, and answering.
Check out our Code of Conduct.









asked 2 days ago









khoshrangkhoshrang

596




596




New contributor




khoshrang is a new contributor to this site. Take care in asking for clarification, commenting, and answering.
Check out our Code of Conduct.





New contributor





khoshrang is a new contributor to this site. Take care in asking for clarification, commenting, and answering.
Check out our Code of Conduct.






khoshrang is a new contributor to this site. Take care in asking for clarification, commenting, and answering.
Check out our Code of Conduct.











  • $begingroup$
    EDIT: ok now i understand the ϕ<π/4 it's because if we put x=0 in z=√ x^2+y^2we will get z=+y and z = -y which explains the ϕ bounds
    $endgroup$
    – khoshrang
    2 days ago
















  • $begingroup$
    EDIT: ok now i understand the ϕ<π/4 it's because if we put x=0 in z=√ x^2+y^2we will get z=+y and z = -y which explains the ϕ bounds
    $endgroup$
    – khoshrang
    2 days ago















$begingroup$
EDIT: ok now i understand the ϕ<π/4 it's because if we put x=0 in z=√ x^2+y^2we will get z=+y and z = -y which explains the ϕ bounds
$endgroup$
– khoshrang
2 days ago




$begingroup$
EDIT: ok now i understand the ϕ<π/4 it's because if we put x=0 in z=√ x^2+y^2we will get z=+y and z = -y which explains the ϕ bounds
$endgroup$
– khoshrang
2 days ago










1 Answer
1






active

oldest

votes


















4












$begingroup$

The plane $z=4$ has equation $r=4sec phi.$ So your limits for $r$ are $0leq r leq 4sec phi.$






share|cite|improve this answer









$endgroup$








  • 1




    $begingroup$
    wow i can't believe how much time i wasted for this only because i was trying with the cylinder "r" concept . thank you
    $endgroup$
    – khoshrang
    2 days ago










Your Answer





StackExchange.ifUsing("editor", function ()
return StackExchange.using("mathjaxEditing", function ()
StackExchange.MarkdownEditor.creationCallbacks.add(function (editor, postfix)
StackExchange.mathjaxEditing.prepareWmdForMathJax(editor, postfix, [["$", "$"], ["\\(","\\)"]]);
);
);
, "mathjax-editing");

StackExchange.ready(function()
var channelOptions =
tags: "".split(" "),
id: "69"
;
initTagRenderer("".split(" "), "".split(" "), channelOptions);

StackExchange.using("externalEditor", function()
// Have to fire editor after snippets, if snippets enabled
if (StackExchange.settings.snippets.snippetsEnabled)
StackExchange.using("snippets", function()
createEditor();
);

else
createEditor();

);

function createEditor()
StackExchange.prepareEditor(
heartbeatType: 'answer',
autoActivateHeartbeat: false,
convertImagesToLinks: true,
noModals: true,
showLowRepImageUploadWarning: true,
reputationToPostImages: 10,
bindNavPrevention: true,
postfix: "",
imageUploader:
brandingHtml: "Powered by u003ca class="icon-imgur-white" href="https://imgur.com/"u003eu003c/au003e",
contentPolicyHtml: "User contributions licensed under u003ca href="https://creativecommons.org/licenses/by-sa/3.0/"u003ecc by-sa 3.0 with attribution requiredu003c/au003e u003ca href="https://stackoverflow.com/legal/content-policy"u003e(content policy)u003c/au003e",
allowUrls: true
,
noCode: true, onDemand: true,
discardSelector: ".discard-answer"
,immediatelyShowMarkdownHelp:true
);



);






khoshrang is a new contributor. Be nice, and check out our Code of Conduct.









draft saved

draft discarded


















StackExchange.ready(
function ()
StackExchange.openid.initPostLogin('.new-post-login', 'https%3a%2f%2fmath.stackexchange.com%2fquestions%2f3142266%2fhow-should-i-solve-this-triple-integral%23new-answer', 'question_page');

);

Post as a guest















Required, but never shown

























1 Answer
1






active

oldest

votes








1 Answer
1






active

oldest

votes









active

oldest

votes






active

oldest

votes









4












$begingroup$

The plane $z=4$ has equation $r=4sec phi.$ So your limits for $r$ are $0leq r leq 4sec phi.$






share|cite|improve this answer









$endgroup$








  • 1




    $begingroup$
    wow i can't believe how much time i wasted for this only because i was trying with the cylinder "r" concept . thank you
    $endgroup$
    – khoshrang
    2 days ago















4












$begingroup$

The plane $z=4$ has equation $r=4sec phi.$ So your limits for $r$ are $0leq r leq 4sec phi.$






share|cite|improve this answer









$endgroup$








  • 1




    $begingroup$
    wow i can't believe how much time i wasted for this only because i was trying with the cylinder "r" concept . thank you
    $endgroup$
    – khoshrang
    2 days ago













4












4








4





$begingroup$

The plane $z=4$ has equation $r=4sec phi.$ So your limits for $r$ are $0leq r leq 4sec phi.$






share|cite|improve this answer









$endgroup$



The plane $z=4$ has equation $r=4sec phi.$ So your limits for $r$ are $0leq r leq 4sec phi.$







share|cite|improve this answer












share|cite|improve this answer



share|cite|improve this answer










answered 2 days ago









B. GoddardB. Goddard

19.6k21442




19.6k21442







  • 1




    $begingroup$
    wow i can't believe how much time i wasted for this only because i was trying with the cylinder "r" concept . thank you
    $endgroup$
    – khoshrang
    2 days ago












  • 1




    $begingroup$
    wow i can't believe how much time i wasted for this only because i was trying with the cylinder "r" concept . thank you
    $endgroup$
    – khoshrang
    2 days ago







1




1




$begingroup$
wow i can't believe how much time i wasted for this only because i was trying with the cylinder "r" concept . thank you
$endgroup$
– khoshrang
2 days ago




$begingroup$
wow i can't believe how much time i wasted for this only because i was trying with the cylinder "r" concept . thank you
$endgroup$
– khoshrang
2 days ago










khoshrang is a new contributor. Be nice, and check out our Code of Conduct.









draft saved

draft discarded


















khoshrang is a new contributor. Be nice, and check out our Code of Conduct.












khoshrang is a new contributor. Be nice, and check out our Code of Conduct.











khoshrang is a new contributor. Be nice, and check out our Code of Conduct.














Thanks for contributing an answer to Mathematics Stack Exchange!


  • Please be sure to answer the question. Provide details and share your research!

But avoid


  • Asking for help, clarification, or responding to other answers.

  • Making statements based on opinion; back them up with references or personal experience.

Use MathJax to format equations. MathJax reference.


To learn more, see our tips on writing great answers.




draft saved


draft discarded














StackExchange.ready(
function ()
StackExchange.openid.initPostLogin('.new-post-login', 'https%3a%2f%2fmath.stackexchange.com%2fquestions%2f3142266%2fhow-should-i-solve-this-triple-integral%23new-answer', 'question_page');

);

Post as a guest















Required, but never shown





















































Required, but never shown














Required, but never shown












Required, but never shown







Required, but never shown

































Required, but never shown














Required, but never shown












Required, but never shown







Required, but never shown







Popular posts from this blog

Lowndes Grove History Architecture References Navigation menu32°48′6″N 79°57′58″W / 32.80167°N 79.96611°W / 32.80167; -79.9661132°48′6″N 79°57′58″W / 32.80167°N 79.96611°W / 32.80167; -79.9661178002500"National Register Information System"Historic houses of South Carolina"Lowndes Grove""+32° 48' 6.00", −79° 57' 58.00""Lowndes Grove, Charleston County (260 St. Margaret St., Charleston)""Lowndes Grove"The Charleston ExpositionIt Happened in South Carolina"Lowndes Grove (House), Saint Margaret Street & Sixth Avenue, Charleston, Charleston County, SC(Photographs)"Plantations of the Carolina Low Countrye

random experiment with two different functions on unit interval Announcing the arrival of Valued Associate #679: Cesar Manara Planned maintenance scheduled April 23, 2019 at 00:00UTC (8:00pm US/Eastern)Random variable and probability space notionsRandom Walk with EdgesFinding functions where the increase over a random interval is Poisson distributedNumber of days until dayCan an observed event in fact be of zero probability?Unit random processmodels of coins and uniform distributionHow to get the number of successes given $n$ trials , probability $P$ and a random variable $X$Absorbing Markov chain in a computer. Is “almost every” turned into always convergence in computer executions?Stopped random walk is not uniformly integrable

How should I support this large drywall patch? Planned maintenance scheduled April 23, 2019 at 00:00UTC (8:00pm US/Eastern) Announcing the arrival of Valued Associate #679: Cesar Manara Unicorn Meta Zoo #1: Why another podcast?How do I cover large gaps in drywall?How do I keep drywall around a patch from crumbling?Can I glue a second layer of drywall?How to patch long strip on drywall?Large drywall patch: how to avoid bulging seams?Drywall Mesh Patch vs. Bulge? To remove or not to remove?How to fix this drywall job?Prep drywall before backsplashWhat's the best way to fix this horrible drywall patch job?Drywall patching using 3M Patch Plus Primer